Đến nội dung

Hình ảnh

$f(x^3+y^3+z^3)=(f(x))^3+(f(y))^3+(f(z))^3$

- - - - - 100hamso

  • Please log in to reply
Chủ đề này có 2 trả lời

#1
namcpnh

namcpnh

    Red Devil

  • Hiệp sỹ
  • 1153 Bài viết

Bài toán 31: Tìm $f:\mathbb{Z}\rightarrow \mathbb{Z}$ thỏa : $f(x^3+y^3+z^3)=(f(x))^3+(f(y))^3+(f(z))^3$.

 

Bài làm : 

 

Bằng pp đồng nhất hệ số thì ta tìm được

$(2n+1)^3=(2n-1)^3+(n+4)^3+(4-n)^3+(-5)^3+(-1)^3$

$(2n+2)^3=(2n-2)^3+(n+8)^3+(8-n)^3+(-10)^3+(-2)^3$

Suy ra mọi số nguyên dương lớn hơn $10$ đều có thể biểu diễn lập phương của nó dưới dạng tổng của $5$ lập phương của các số nguyên khác có giá trị tuyệt đối nhỏ hơn nó

Trong giả thiết đã cho thay $x=y=z=0$ thì ta được $f(0)=3f^3(0)$ nên $f(0)=0$ (vì $f:\mathbb{Z}\rightarrow \mathbb{Z}$ )

Thay $y=z=0$, ta có $f^3(x)=(f(x))^3$

Thay $y=-z$, ta có $(f(x))^3=f^3(x)+f^3(y)+f^3(-y)\Rightarrow f^3(y)=-(f(y))^3\Rightarrow f(y)=-f(-y)$

Ta sẽ chứng minh bằng quy nạp:$\forall k\in \mathbb{Z}, f(k)=kf^3(1)$

Do $f(y)=-f(-y)$ nên ta chỉ cần xét $k\in \mathbb{N}$

Với $k=0$ thì khẳng định hiển nhiên đúng

Với $k=1$:Thay $x=1, y=z=0$ thì $f(1)=f^3(1)$

Với $k=2$:Thay $x=y=1, z=0$, ta có $f(2)=2f^3(1)$

Với $k=3$:Thay $x=y=z=1$, ta có $f(3)=3f^3(1)$

Với $k=6$:Thay $x=2, y=z=-1$, ta có $f(6)=f^3(2)-2f^3(1)=6f^3(1)$
Với $k=7$:Thay $x=2, y=-1, z=0$ ta có $f(7)=f^3(2)-f^3(1)=7f^3(1)$
Với $k=8$:Thay $x=2,y=z=0$ ta có $f(8)=(2f(1))^3=8f^3(1)$

Với $k=9$:Thay $x=2, y=1,z=0$ ta có $f(9)=f^3(2)+f^3(1)=9f^3(1)$

Với $k=5$:Ta có $5^3=3^3+6^3+2^3+(-5)^3+(-1)^3\Rightarrow 5^3+5^3+1^3=3^3+6^3+2^3\Rightarrow f(5^3+5^3+1^3)=f(3^3+6^3+2^3)\Rightarrow 2f^3(5)+f^3(1)=f^3(3)+f^3(6)+f^3(2)=251f^9(1)\Rightarrow f(5)=5f^3(1)$
Với $k=4$:Làm tương tự $k=5$, ta có $f(4)=4f^3(1)$

Do đó khẳng định đúng với mọi $k$ mà $k< 10$
Với $k> 10$ 

Xét $k=2n+1$. Ta có $(2n+1)^3+5^3+(-1)^3=(2n-1)^3+(n+4)^3+(4-n)^3$

Suy ra $f^3(2n+1)+f^3(5)+f^3(1)=f^3(2n-1)+f^3(n+4)+f^3(4-n)$
Theo nx ban đầu, vì $k> 10$ nên ở đẳng thức bên trên ta thấy ngay các số $2n-1, n+4, 4-n, -5, -1$ đều có giá trị tuyệt đối nhỏ hơn $2n+1$, nên theo giả thiết quy nạp thì $f(5)=5f^3(1), f(2n-1)=(2n-1)f^3(1), f(n+4)=(n+4)f^3(1), f(4-n)=(4-n)f^3(1)$

Suy ra $f(2n+1)=(2n+1)f^3(1)$
Với $k=2n+2$ ta cũng làm tương tự và có $f(2n+2)=(2n+2)f^3(1)$

Vậy khẳng định ban đầu đúng với mọi $k\in \mathbb{N}$

Từ $f(1)=f^3(1)$ ta có $f(1)=0$ hặoc $f(1)=1$ hoặc $f(1)=-1$
Suy ra $f(k)=0\forall k\in \mathbb{Z}$, $f(k)=k\forall k\in \mathbb{Z}$, $f(k)=-k\forall k\in \mathbb{Z}$

Thử lại thấy cả $3$ hàm thỏa mãn

Cách tìm ra 2 đẳng thức kia thì e làm như thế này:

Với $k=2n+1$:Ta tìm 1 đẳng thức mà $2k+1$ có GTTĐ lớn nhất và để khử $8k^3$, ta chọn $-(2k-1)^3$. Ta chọn tiếp $2$ biểu thức có chứa lượng $k$ như nhau, cụ thể là $(pk+q)^3$ và $(pk-q)^3$

Khi đó $(2k+1)^3-(2k-1)^3=(pk+q)^3+(pk-q)^3+c^3+d^3$

Suy ra $k^2(24-6p^2q)+(2-2q^3-c^3-d^3)=0$

Chọn các số thích hợp sao cho $p^2q=4, 2q^3+c^3+d^3=2$, với $p\leq 2$

Bằng cách thử, ta có $p=1, q=4$ nên $c^3+d^3=-126$, suy ra $c=-5, d=-1$

Cái còn lại tương tự


Bài viết đã được chỉnh sửa nội dung bởi namcpnh: 01-12-2013 - 11:23

Cùng chung sức làm chuyên đề hay cho diễn đàn tại :

Dãy số-giới hạn, Đa thức , Hình học , Phương trình hàm , PT-HPT-BPT , Số học.

Wolframalpha đây


#2
ntuan5

ntuan5

    Hạ sĩ

  • Thành viên
  • 93 Bài viết

Bài này chắc là sử dụng quy nạp với hằng đẳng thức ( $f(n)=nf(1)$), dùng phương pháp hằng số bất định ta được:

 Với $n=2k+1$ : $(2k+1)^3+5^3+1^3=(k-4)^3+(4-k)^3+(2k-1)^3$.

 Với n chẵn để sử dụng được phương pháp trên ( khi đưa về bậc thấp, khử được bậc 1 có hai hằng số ở một vế, thì chon $n=2k+2$). Biểu diễn lại $(2k+2)^=(2k-2)+(a-k)^3+(a+k)^3-...-...$. Sau này chắc cũng tìm được hằng số ( sao mình tính cứ nhầm). Cuối cùng là thử lại.


Bài viết đã được chỉnh sửa nội dung bởi ntuan5: 28-05-2013 - 19:47

  • LNH yêu thích

#3
vutuanhien

vutuanhien

    Thiếu úy

  • ĐHV Toán Cao cấp
  • 690 Bài viết

Bài toán 31: Tìm $f:\mathbb{Z}\rightarrow \mathbb{Z}$ thỏa : $f(x^3+y^3+z^3)=(f(x))^3+(f(y))^3+(f(z))^3$

Không thấy ai giải bài này nên e nhảy vào giải thử, hi vọng đúng

Ý tưởng là dựa vào đẳng thức mà ntuan5 đã nói đến :P

Bằng pp đồng nhất hệ số thì ta tìm được

$(2n+1)^3=(2n-1)^3+(n+4)^3+(4-n)^3+(-5)^3+(-1)^3$

$(2n+2)^3=(2n-2)^3+(n+8)^3+(8-n)^3+(-10)^3+(-2)^3$

Suy ra mọi số nguyên dương lớn hơn $10$ đều có thể biểu diễn lập phương của nó dưới dạng tổng của $5$ lập phương của các số nguyên khác có giá trị tuyệt đối nhỏ hơn nó

Trong giả thiết đã cho thay $x=y=z=0$ thì ta được $f(0)=3f^3(0)$ nên $f(0)=0$ (vì $f:\mathbb{Z}\rightarrow \mathbb{Z}$ )

Thay $y=z=0$, ta có $f^3(x)=(f(x))^3$

Thay $y=-z$, ta có $(f(x))^3=f^3(x)+f^3(y)+f^3(-y)\Rightarrow f^3(y)=-(f(y))^3\Rightarrow f(y)=-f(-y)$

Ta sẽ chứng minh bằng quy nạp:$\forall k\in \mathbb{Z}, f(k)=kf^3(1)$

Do $f(y)=-f(-y)$ nên ta chỉ cần xét $k\in \mathbb{N}$

Với $k=0$ thì khẳng định hiển nhiên đúng

Với $k=1$:Thay $x=1, y=z=0$ thì $f(1)=f^3(1)$

Với $k=2$:Thay $x=y=1, z=0$, ta có $f(2)=2f^3(1)$

Với $k=3$:Thay $x=y=z=1$, ta có $f(3)=3f^3(1)$

Với $k=6$:Thay $x=2, y=z=-1$, ta có $f(6)=f^3(2)-2f^3(1)=6f^3(1)$
Với $k=7$:Thay $x=2, y=-1, z=0$ ta có $f(7)=f^3(2)-f^3(1)=7f^3(1)$
Với $k=8$:Thay $x=2,y=z=0$ ta có $f(8)=(2f(1))^3=8f^3(1)$

Với $k=9$:Thay $x=2, y=1,z=0$ ta có $f(9)=f^3(2)+f^3(1)=9f^3(1)$

Với $k=5$:Ta có $5^3=3^3+6^3+2^3+(-5)^3+(-1)^3\Rightarrow 5^3+5^3+1^3=3^3+6^3+2^3\Rightarrow f(5^3+5^3+1^3)=f(3^3+6^3+2^3)\Rightarrow 2f^3(5)+f^3(1)=f^3(3)+f^3(6)+f^3(2)=251f^9(1)\Rightarrow f(5)=5f^3(1)$
Với $k=4$:Làm tương tự $k=5$, ta có $f(4)=4f^3(1)$

Do đó khẳng định đúng với mọi $k$ mà $k< 10$
Với $k> 10$ 

Xét $k=2n+1$. Ta có $(2n+1)^3+5^3+(-1)^3=(2n-1)^3+(n+4)^3+(4-n)^3$

Suy ra $f^3(2n+1)+f^3(5)+f^3(1)=f^3(2n-1)+f^3(n+4)+f^3(4-n)$
Theo nx ban đầu, vì $k> 10$ nên ở đẳng thức bên trên ta thấy ngay các số $2n-1, n+4, 4-n, -5, -1$ đều có giá trị tuyệt đối nhỏ hơn $2n+1$, nên theo giả thiết quy nạp thì $f(5)=5f^3(1), f(2n-1)=(2n-1)f^3(1), f(n+4)=(n+4)f^3(1), f(4-n)=(4-n)f^3(1)$

Suy ra $f(2n+1)=(2n+1)f^3(1)$
Với $k=2n+2$ ta cũng làm tương tự và có $f(2n+2)=(2n+2)f^3(1)$

Vậy khẳng định ban đầu đúng với mọi $k\in \mathbb{N}$

Từ $f(1)=f^3(1)$ ta có $f(1)=0$ hặoc $f(1)=1$ hoặc $f(1)=-1$
Suy ra $f(k)=0\forall k\in \mathbb{Z}$, $f(k)=k\forall k\in \mathbb{Z}$, $f(k)=-k\forall k\in \mathbb{Z}$

Thử lại thấy cả $3$ hàm thỏa mãn

Cách tìm ra 2 đẳng thức kia thì e làm như thế này:

Với $k=2n+1$:Ta tìm 1 đẳng thức mà $2k+1$ có GTTĐ lớn nhất và để khử $8k^3$, ta chọn $-(2k-1)^3$. Ta chọn tiếp $2$ biểu thức có chứa lượng $k$ như nhau, cụ thể là $(pk+q)^3$ và $(pk-q)^3$

Khi đó $(2k+1)^3-(2k-1)^3=(pk+q)^3+(pk-q)^3+c^3+d^3$

Suy ra $k^2(24-6p^2q)+(2-2q^3-c^3-d^3)=0$

Chọn các số thích hợp sao cho $p^2q=4, 2q^3+c^3+d^3=2$, với $p\leq 2$

Bằng cách thử, ta có $p=1, q=4$ nên $c^3+d^3=-126$, suy ra $c=-5, d=-1$

Cái còn lại tương tự


"The first analogy that came to my mind is of immersing the nut in some softening liquid, and why not simply water? From time to time you rub so the liquid penetrates better, and otherwise you let time pass. The shell becomes more flexible through weeks and months—when the time is ripe, hand pressure is enough, the shell opens like a perfectly ripened avocado!" - Grothendieck






Được gắn nhãn với một hoặc nhiều trong số những từ khóa sau: 100hamso

0 người đang xem chủ đề

0 thành viên, 0 khách, 0 thành viên ẩn danh